0 Daumen
1,5k Aufrufe

$$ \sum _{ i=1 }^{ n } i^{ 3 }=(\sum _{ i=1 }^{ n } i)^{ 2 }=(\frac { n(n+1) }{ 2 } )^{ ^{ 2 } } \forall n \in N $$

Induktionanfang: $$ n = 1 \\ 1^3 = (\frac{1(1+1)}{2})^2 \\ 1 = (\frac{2}{2})^2 = \underline{1} $$

Induktionsschritt: $$ n \rightarrow n+1 $$

Vorausetzung: $$ \sum _{ i=1 }^{ n } i^{ 3 }=(\sum _{ i=1 }^{ n } i)^{ 2 }=(\frac { n(n+1) }{ 2 } )^{ ^{ 2 } }  $$

Behauptung:

$$ \sum _{ i=1 }^{ n+1 } i^{ 3 }=(\frac { (n+1)(n+2) }{ 2 } )^{ ^{ 2 } }=(\frac { n^{ 2 }+3n+2 }{ 2 } )^{ ^{ 2 } }\\ =\frac { 1 }{ 4 } (n^{ 2 }+3n+2)^{ 2 }\\ =\frac { n^{ 4 } }{ 4 } +\frac { 3n^{ 3 } }{ 4 } +\frac { 13n^{ 2 } }{ 2 } +\frac { 3n }{ 2 } +1 $$

Beweis:

$$ \sum _{ i=1 }^{ n+1 } i^{ 3 }=\sum _{ i=1 }^{ n } i^{ 3 }+(n+1)^{ 3 }\\ =(\frac { n(n+1) }{ 2 } )^{ ^{ 2 } }+(n+1)^{ 3 }=\frac { 1 }{ 4 } (n^{ 2 }+n)^{ 2 }+(n+1)^{ 3 }\\ =\frac { 1 }{ 4 } (n^{ 4 }+2n^{ 2 }+n^{ 2 })+(n^{ 3 }+3n^{ 2 }+3n+1) $$

Wenn ich jetzt denn hinteren Summanden mit $$ \frac{1}{4} $$ erweitere, erhalte ich.

$$ =\frac { 1 }{ 4 } (n^4+3n^3+4n^2+3n+1) $$

Ich finde den Fehler nicht, kann mir da jemand bitte weiter helfen. Danke

Avatar von

1 Antwort

0 Daumen

Induktionsschritt: Nun zeigen wir, dass es für n + 1 gilt, unter der Annahme dass es für n gilt.

 

Σ (i = 1 bis n) (i^3) + (n + 1)^3 = Σ (i = 1 bis n + 1) (i^3)

(n·(n + 1)/2)^2 + (n + 1)^3 = ((n + 1)·(n + 2)/2)^2

(n^2/2 + n/2)^2 + (n + 1)^3 = (n^2/2 + 3·n/2 + 1)^2

(n^4/4 + n^3/2 + n^2/4) + (n^3 + 3·n^2 + 3·n + 1) = (n^4/4 + 3·n^3/2 + 13·n^2/4 + 3·n + 1)

n^4/4 + 3·n^3/2 + 13·n^2/4 + 3·n + 1 = n^4/4 + 3·n^3/2 + 13·n^2/4 + 3·n + 1

 

1/4·(n^4 + 6·n^3 + 13·n^2 + 12·n + 4) = 1/4·(n^4 + 6·n^3 + 13·n^2 + 12·n + 4)

Avatar von 477 k 🚀

Ein anderes Problem?

Stell deine Frage

Willkommen bei der Mathelounge! Stell deine Frage einfach und kostenlos

x
Made by a lovely community